(Multivariable Calc) Given four points, find a sphere.help me.

Click For Summary
To find the center and radius of a sphere determined by four points, the equation of a sphere can be utilized. The user initially struggled with using the determinant formula related to a 5x5 matrix but later realized that substituting the coordinates into the sphere's equation would yield a system of equations. By solving this system through row reduction, the values for the center (h, k, l) and radius (r) can be determined. The discussion highlights the importance of correctly setting up the equations and understanding the variables involved. Ultimately, the problem was resolved by applying these methods effectively.
Vampire
Messages
11
Reaction score
0

Homework Statement


Find the radius and center for the sphere determined by the four points
(3,2,-1), (5,-2,1), (-3,1,2), (0,2,4).
Give the exact location of the center but approximate the radius to the nearest 0.1.

Homework Equations


Equation of a sphere: x2 + y2 + z2 + Gx +Hy +Iz +J = 0
or C(h,k,l) (x-h)2 + (y-k)2 + (z-l)2 - r2 = 0

There's also a 5x5 matrix with the determinant equal to zero, but I have no idea how to use it (seen at line 29 here: http://mathworld.wolfram.com/Sphere.html). I believe this is beyond 12.1 in multivariable calculus.

The Attempt at a Solution


I tried to use the determinant formula, but I got lost as I don't even know how to use it.

Is there any other way?

EDIT: Solved. Thank you.
 
Last edited:
Physics news on Phys.org
(x-h)2 + (y-k)2 + (z-l)2 - r2 = 0

Using this, I'd just substitute the 4 coordinates to get me 4 equations with 4 unknowns in them. Then just use row reduction to get h,k,l and r
 
So I can substitute, then treat the different lines as a system of equations?
 
Vampire said:
So I can substitute, then treat the different lines as a system of equations?

well your variables would be h2,k2,l2,r2 and not h,k,l,r like I previously said.
 
Alright I will try that. Thank you very much.
 
Question: A clock's minute hand has length 4 and its hour hand has length 3. What is the distance between the tips at the moment when it is increasing most rapidly?(Putnam Exam Question) Answer: Making assumption that both the hands moves at constant angular velocities, the answer is ## \sqrt{7} .## But don't you think this assumption is somewhat doubtful and wrong?

Similar threads

Replies
1
Views
2K
  • · Replies 11 ·
Replies
11
Views
12K
  • · Replies 7 ·
Replies
7
Views
3K
Replies
2
Views
13K
  • · Replies 6 ·
Replies
6
Views
9K
  • · Replies 10 ·
Replies
10
Views
2K
  • · Replies 2 ·
Replies
2
Views
7K
  • · Replies 1 ·
Replies
1
Views
2K
Replies
1
Views
9K
  • · Replies 11 ·
Replies
11
Views
7K